Вы находитесь на странице: 1из 13

RT 48+ GMAT MATH QUIZ #1

Ever wondered what it feels like to take the GMAT and score in the top 10% on the quantitative section? Below is a 37-
question math workout to give you the experience of the kinds of questions you are likely to see if you ace the math portion
of the test. If you are aiming for a perfect score, this is a great opportunity to practice your pacing and accuracy, as well as to
work out on the kind of questions ETS is throwing at its top scorers these days. If you haven’t reached the top level yet, this
is a great way to stretch yourself and to practice teasing your brain on the very hardest questions out there.

1. Stock Y starts at $5 per share and then −1


 
increases to $6 per share. What is the percent  
 1  =
increase of the price per share of stock Y? 5.
 1 + 1 
(A) 15%  −1 
(B) 16.6% 6 7 −1 
(C) 18.3%
(D) 20% 1
(A)
(E) 83.3% 42
1
(B)
2. If 5 25 × 413 = 2 ×10 k , what is the value of k? 13
(A) 13 42
(B) 24 (C)
13
(C) 25
(D) 13
(D) 26 (E) 42
(E) 38

m−n
3. What is the value of = 6. If there are 400 women at College X and 200
m 2 − n2 students live off- campus, how many students
(1) m2 – n2 = 40 does College X have?
(2) m + n = 10 (1) 120 women live off-campus.
(2) There are half as many men at College X as
4. If the two sets have an equal number of there are students who live on-campus.
numbers, is the mean of set Q lower than the
mean of set P?

(1) Set Q consists of consecutive even integers


and set P of consecutive odd integers.
(2) The median of Q is higher than the mean of P.

1
The Princeton Review Management, LLC 2002
7. m
10. Is mn < ?
n
C (1) mn is a positive integer.
B (2) n is a negative number.

A 1
11. If r ≠ 0 and r * is defined as − 1 , what
r
is the value of − (r *) + (r − 1) * ?
1
(A)
If the figure above is a cube, and A, B and C are r −r
2

midpoints on the edges of the cube, what is the r2 + r −1


measure of ∠ ABC? (B)
r2 − r
(A) 30
2r 2 + 2r + 1
(B) 45 (C)
(C) 60 r − r2
(D) 90 2 − 3r
(D)
(E) 120 r
2r − 1
(E)
r2 − r
8. Goldenrod and No Hope are in a horse race
with 6 contestants. How many different
arrangements of finishes are there if No Hope 12.
always finishes before Goldenrod and if all of the
horses finish the race?
(A) 720
(B) 360
(C) 120
(D) 24
(E) 21

4 Which of the following is a possible equation for


9. 4 80 + = ? the above graph?
9+4 5
A) x 3
4 5 B) x 3 -1
(A)
9 C) 3x3 + 2x
(B) 2 D) 3x 3 – 2x
(C) 6 E) x 3 + 3x2 – x + 2
4 5 +2
(D)
3+ 2 5
(E) 36

2
The Princeton Review Management, LLC 2002
13. If 1 < a < b < c < d, what is the value of d? 17.

(1) a + b + c + d = 170
(2) a, b, c, and d are all positive integers in the
set of numbers that can be written in the x° z°
form of 2n , where n is also a positive w°
integer.
What is the value of y?

14. A committee of 6 is chosen from 8 men and 5 (1) w – x = 65


women so as to contain at least 2 men and 3
(2) x + z = 115
women. How many different committees could
be formed if two of the men refuse to serve
together?
(A) 3510 18. Given that x& = x 2 + 3, what is the value of
(B) 2620 integer k if there is a remainder of 3 when k& is
(C) 1404 divided by 4?
(D) 700
(E) 635 (1) 16 < k& < 37
(2) k is a factor of 24.
 a  b  c   b  c 
15. Is     >    ?
 d  e  f   d  e  19. If xy ≠ 0 , which quadrant is point (x, y) in?

(1) bc > de 1− x
(2) a > f (1) >0
1− y
(2) x + y = 20
16. Six people are on an elevator that stops at
exactly 6 floors. What is the probability that
exactly one person will push the button for each
20. The value of â is the product of all positive,
floor?
odd integers less then a but greater than 0. If
6!
(A) 6 e = 16 and o = 14 what is the largest prime factor
6 of ê + 2ô?
66 (A) 23
(B)
6! (B) 19
6 (C) 17
(C) (D) 13
6!
6 (E) 11
(D) 6
6
1
(E) 6
6

3
The Princeton Review Management, LLC 2002
21. Amy is planting rose bushes in her garden. 24. The probability that it will rain in NYC on
Her pink, red and white bushes are not planted in any given day in July is 40%. What is the
a straight line. If the pink bush is 5 yards from probability that it will rain in NYC on exactly 2
the red, and the red bush is 11 yards from the days out of 5 in July?
white. Which of the following is a possible 2 2 33
distance of the white bush from the pink? (A)
55
I. 5
II. 6 2 3 33
(B)
III. 7 55
2 2 33
(C)
(A) None of the above 54
(B) III only 23 32
(C) II and III (D)
(D) I and II 54
(E) I, II, and III 2 3 33
(E)
54

22. In quadrilateral WXYZ, is XY perpendicular to


YZ?
( )
25. If ab = – 1 and a = 4 − 17 , what is the
value of b?
1
(A)
(1) WXY is a right angle 4 − 17
(2) WZY is a right angle 4 − 17
(B)
4
23. A tourist has travelers’ checks in $20 and (C) 4 + 17
$100 denominations. How many $20 checks are (D) 1 + 17
there? (E) − 4 − 17

(1) If half of the $20 checks is spent, the


remaining amount is $520. 26. A circle is inscribed in equilateral triangle
ABC so that point D lies on the circle and on line
(2) The total value of the checks is $740. segment AC and point E lies on the circle and on
line segment AB. If line segment AB = 6, what is
the area of the figure created by line segments
AD, AE and minor arc DE?
9
(A) 3 3 − π
4
(B) 3 3 − π
(C) 6 3 − 3π
(D) 9 3 − 3π
(E) It cannot be determined from the information
given.

4
The Princeton Review Management, LLC 2002
27. There are 16 stones in a bowl: 4 are painted 31. In corporation X, the finance division has an
orange, 3 green and the rest purple. What is the average annual salary of $102,000. The
probability of getting first an orange and then a consulting division has an average annual salary
purple stone if you choose two stones at random? of $73,000. What is the average annual salary of
9 the two divisions combined?
(A)
80
3 (1) The two divisions have a total of 42
(B)
64 workers.
9
(C) (2) There are twice as many employees in the
64
1 consulting division as there are in the
(D) finance division.
20
3
(E)
20 (−1) n
32. If in sequence R, rn +1 − rn = , then what
n
is the order, from smallest to largest of r5 , r6 and
28. If f (a + b) = f (a) + f (b), then which of the r7 ?
following could be f (x) for all distinct values of a (A) r5 < r6 < r7
and b? (B) r6 < r5 < r7
A) x 2 + 1 (C) r5 < r7 < r6
B) 2 x (D) r6 < r7 < r5
C) x − 4 (E) r7 < r5 < r6
5
D)
x
E) − 7 x 33. There are 3 black balls and 7 white balls in a
box. If two balls are chosen randomly, what is
the probability that at least one will be black?
29. If mn ≠ 0 , what is the ratio of m to n2 ? 1
(A)
7 15
(1) The ratio of m2 to 1 is 7
5 (B)
7 30
(2) The ratio of m2 to n is 7
5 (C)
15
8
30. Jon’s cat consumes an average of 2.5 lbs of (D)
food per day. Yesterday, the cat ate only .5 lbs of 15
23
food. Approximately what percent of the average (E)
weekly food did the cat eat yesterday? 30
(A) 28%
(B) 2.8%
(C) 2% 34. If x = 274 – 270 , what is the largest prime
(D) .28% factor of x?
(E) .028% (A) 2
(B) 3
(C) 5
(D) 7
(E) 11

5
The Princeton Review Management, LLC 2002
35. A distinct three-digit pin number is
distributed to each member of a video club. All
digits 0-9 are allowed, and digits can be repeated
to create distinct pin numbers. If 327 pin
numbers are not used, how many members
belong to the video club?
(A) 1000
(B) 940
(C) 673
(D) 613
(E) 402

0.08 −5
36. =?
0.04 − 4
5
(A)
64
5
(B)
32
25
(C)
32
32
(D)
25
(E) 50

37. If x = 6 xy − 9 y 2 , what is the value of x in


terms of y?
(A) 3 y
1
(B) − y
3
y2
(C)
3 + 6y
(D) − 3 y
(E) 6 y − 9 y 2

6
The Princeton Review Management, LLC 2002
Answer key:

1.
2.
3.
4.
5.
6.
7.
8.
9.
10.
11.
12.
13.
14.
15.
16.
17.
18.
19.
20.
21.
22.
23.
24.
25.
26.
27.
28.
29.
30.
31.
32.
33.
34.
35.
36.
37.

7
The Princeton Review Management, LLC 2002
1. (D). This is a basic percent increase 5. (B). This one looks a lot scarier than it
question. The formula you should use is actually is. If you know your exponent
difference rules well, it should be no problem:
× 100 = %change . That works −1
original  
  −1 −1
6 −5 1  1   1  1
out to be × 100 = × 100 = 20% . =  =   = 13 .
5 5  1 + 1  6+7  13 
 −1 
6 7 −1 
2. (C). When you have questions that deal
with variables in the exponent, your goal 6. (C). This is a question that should be set
should always be to convert all your base up with a groups grid:
numbers to the same number, if possible.
Dealing with the left side of the equation From Statement 1:
first, it should look as follows:
5 25 × 413 = 5 25 × (2 2 ) =
13
Women Men Total
Off- 120 200
5 25 × 226 =
campus
(5 × 2)25 × 2 = On-
2 × 10 25 campus
therefore k = 25 . Total 400 Total

3. (B). Remember to rewrite the expression from statement 2:


m 2 − n 2 as (m + n )(m − n ) , whenever you
Women Men Total
see it. If you do that, then it is easy to see
Off- 200
that the piece of the puzzle you are campus
looking for is the value of (m + n ) . On- (2) x
Statement 2 gives it to you, but statement campus
1 does not. Total 400 (2) Total
½ x
4. (C). For statement 1, it is easy to come up Therefore you can set up 2 equations that allow
with examples that will give you a "yes" you to solve for x.
and those that will give you a "no": if you
plug in Set P {1, 3, 5} and Set Q {10, 12,
14} you get a "no", but if you plug in Set
P {11, 13, 15} and Set Q {2, 4, 6}, you
will get a "yes". For statement 2, it gets a
little bit tricky, but it is still possible to get
both answers: if you plug in Set P {1, 3,
5} and Set Q {10, 12, 14} you still get a
"no", but if you keep Set P the same and
change Set Q to {-100, 12, 14} the median
of Q is higher than the mean of P, but Q's
mean is less than P's, so you get a "yes".
When you combine the information,
however, with consecutive sets the mean
is equal to the median, so you can only
come up with "no".

8
The Princeton Review Management, LLC 2002
7. (C). The key to this question, as with scenarios will give us a "no". Statement 1
many geometry questions, is to draw an rules out one but not the other variable
additional line on the diagram. If you being negative, but it does not address the
draw a line between points A and C, you issue of n being a fraction less than one.
get an equilateral triangle. Statement 2 does even less, since we do
not know if m < 0 . Together, the
8. (B). Start out by finding the total number statements still do not tell us if 0 < n < 1 .
of possible runner finish arrangements:
6 × 5 × 4 × 3 × 2 × 1 = 720 . Of those 11. (A). As always when you have variables
arrangements, half of them have No Hope in your answer choices, you want to plug
beating Goldenrod and the other half have in. A good number to choose is r = 2. If
Goldenrod winning. Since Goldenrod you plug it into the question correctly,
never comes in before No Hope, the your target answer should be 1/2. Only
correct answer is  720 = 360 .
1 answer choice (A) gives you 1/2 when
2 you plug in 2 for r.

12. (D). Use the points you can identify to


9. (E). This one is a beast if you do the
simplification. On the other hand, it can eliminate answer choices. Since the curve
be solved quite easily if you ballpark. If passes through the origin, when x = 0, the
result should equal 0 also. By that logic,
you round 80 up to 81, and 5 down to 4,
you can cross off (B) and (E). Then, in
4
you get: 4( 9) + which reduces order to have the curve dip below the x-
9 + 4( 2) axis to the right of the y-axis as it does,
to something very close to 6, the correct some positive values of x must result in a
answer. To prove it mathematically, the negative value for the expression. The
steps you take look like this: only remaining answer that could ever do
4 80 +
4
=
that is the correct answer, (D).
9+4 5
 
4  80 +
1
 = 13. (C). Statement 1 is certainly not enough
 9 + 4 5
information on its own. Nor is statement
 1 
44 5 +  = 2--it simply tells you that all of the
 9+4 5
variables are exponents of 2 (i.e. 2, 4, 8,
4 
( )
 4 5 9 + 4 5 +1 
 = 16, etc.). Together, however, you need to
9+4 5 
  slow down and watch out. Is there only
 36 5 + 80 + 1  one possible value for d? Yes. First list
4  =
 9+4 5  all exponents of 2 less than 170: 2, 4, 8,
 4 5 +9 16, 32, 64, 128. Is there any way to
4 × 9  =
 9+4 5  combine four of the numbers to add up to
36 170 without using 128? If you add up 8 +
16 + 32 + 64, you will see that even that is
10. (E). As with all yes/no data sufficiency, it too small. Therefore, d must equal 128.
is a good idea to get some sense of what
kind of numbers will give you a "yes" and 14. (E). This question has two wrinkles: we
which will give you a "no" before you don't know exactly how many men and
look at the statements. In this question, women are serving on the committee and
m we have some illegal combinations.
mn < when one or the other variable is
n Simplest thing to do would be to
negative or when 0 < n < 1 . All other ballpark. There are two ways to distribute

9
The Princeton Review Management, LLC 2002
the committee slots between men and a
women: 3 of each or 2 me n and 4 women. above inequa lity to hold true, > 1 if the
f
We must treat the two scenarios right side of the inequality is positive,
separately. If there are 3 of each sex on a
the committee, then the total number of < 1 if it is negative. To see how this
possible combinations given this scenario f

works, set    = −1 and then 1.


is found as follows: b c
8 • 7 •6 5• 4 • 3  d  e 
• = 560 . . If there are 2
3 • 2 • 1 3 • 2 •1 Statement 2 ends up telling us very little,
men and 4 wo men, then the total number a
of possible combinations: because we can choose values for that
f
8 • 7 5 • 4 • 3• 2
• = 140. Together you result in numbers on either side of 1.
2 •1 4 • 3 • 2 • 1 a
have a total of 700, since your answer ( < 1 when one or both variables is
excludes some of these options the correct f
answer is smaller than 700. To actually negative). Taking both statements
calculate how the number of people lets together does little to resolve any of these
look at the 3 men 3 women situation. issues.
First you would have to deal with the two
separate possible scenarios; the scenario 16. (A). For probability questions, the
where neither of the refusers are on the # favorable
equation you should use is .
committee and the scenario where exactly # total
one is on the committee. Where neither is Here you should use your knowledge of
on the committee the number of permutations to find the elements of the
6 •5• 4 5 • 4 • 3 formula. There are 6! ways that one
combinations is • .
3• 2 •1 3 • 2 • 1 person is going to each floor, and each of
Where exactly one of the refusers is on 6 people have 6 choices as to which
2 •6• 5 5 • 4 • 3 button to push, which means that there are
the committee is • . Then
2 •1 3•2 •1 66 total ways people's destinations are
calculate the combinations for the 2 men 3 distributed.
women.
17. (D). As with all questions dealing the
15. (E). Since this is a yes/no type data angles of a triangle, here the key to the
sufficiency question, take stock of the question is remembering that the interior
question before you dive into the angles of a triangle add up to 180. For
statements. To understand what is really statement 1, you also know that w + z =
being asked of you, it is helpful to 180 or z = 180 - w. Plug that into the
rearrange the terms in the left side of the expression x + y + z = 180 and you get x
inequality to match better those on the + y + 180 - w = 180. Simplify and solve
 a  b  c   b  c  for y, and you get y = w - x. Therefore,
right:     >    . When you statement 1 is sufficient. To evaluate
 f  d  e   d  e  statement 2, plug 115 in for x + z in the
do that rearrangement, it becomes clear equation x + y + z = 180. When you do
that the only difference between the two that, you get an equation with only one
a variable, which can always be solved.
sides is the term. At this point, it
f
becomes clear that statement 1 does 18. (A). In statement 1, there are two possible
nothing to answer the question. For the values for k&: 19 and 28. Only 19 has a
10
The Princeton Review Management, LLC 2002
remainder of 3 when divided by 4, statements don’t resolve it because it
however, so statement 1 is sufficient. All could be a square or it could be a shape
even values of k will have a remainder of with angles 90, 90, 110 and 80 (try to
3 when divided by 4 (can you explain why draw this you will see its possible).
this is so?), so statement 2 does not give
you enough information. 23. (C). Statement 1 can be expressed as
100H + 0.5 (20T) = 520 (where H =
19. (C). This question is really asking us if number of $100 checks and T = number
we know for sure whether x and y are of $20 checks). There are 2 variables and
positive or negative. Statement 1 is not only one equation, so on its own,
sufficient because (x, y) could equal statement 1 is not sufficient. Statement 2
points in all four quadrants such as (2, 2), can be expressed as 100H + 20T = 740.
(-1, -1), (-1, 0.5), (0.5, -1). Statement 2 is This also is not sufficient on its own.
not sufficient because again, one can find Together, however, the statements are
points in all quadrants but III that would sufficient, since you have two equations
work: (10, 10), (-1, 21), (21, -1). and two unknowns.
Together, however, it is sufficient,
because the only quadrant that now works 24. (E). First find the probability that it will
is I (points such as (10, 10) or (1, 19)). rain on one arrangement of two rainy days
and three sunny days (i.e. it will rain the
20. (C). This function should remind you of first two days and then not rain for the last
factorials, and indeed, the expression ê + three days). This works out as follows:
2ô can be rewritten as 4 4 6 6 6 2 2 33
(15 •13 •11• 9...) + 2(13 •11• 9...) . (Note × × × × = 5 . Now find
10 10 10 10 10 5
that the function is defined as the product how many ways there are to arrange 2
of all even numbers less than a, not less rainy days and 3 sunny days:
than or equal to a.) The common factors 5 × 4 × 3 × 2 ×1
of the two terms can be factored out, so = 10. Now multiply the
the expression is rewritten as 2 × 1× 3 × 2 × 1
probability of any one arrangement
(13 • 11• 9...)(15 + 2) . The largest prime happening times the number of
factor is 17. arrangements and you get
2 23 3 23 3 3
21. (B). This is really a question of × 10 = .
impossible triangles. Only statement III, 55 54
7, satisfies the condition that the sum of
the distance of any two sides of a triangle 25. (C). If you plug in the choices, starting
must be larger than the distance of the with the simplest of the choices and math
third side. that seems ugly will become very easy.
Answer (C), which is one of the easier
22. (E). If you choose to draw the choices, when plugged in should remind
quadrilateral, you must realize that you you of the quadratics, thinking of the
don’t know whether it’s a regular shape GMATs 3 favorite quadratic equations.
such as a parallelogram or whether it’s a Here the one we need is x 2 – y2 .
stranger shape, also when you label points
do so clockwise. Statement 1 only tells 26. (B). In general terms the area of the
about one angle so its insufficient since portion you are looking for can be
the others could be anything. Statement 2 triangle − circle
described as . The area
has the exact same problem. Together the 3
11
The Princeton Review Management, LLC 2002
of the triangle is fairly straight forward: 29. (C). Statement 1 provides no info about n
the base equals 6 and the height equals so its insufficient. Statement two almost
3 3 , so the area equals 9 3 . In order to gives you a ratio of m to n (when m
find the area of the circle, we need to find is 7 ≈ 2.64 , n is 5, and all multiples of
the radius, and this is a bit tricky, but if this), however because you are told about
you draw the right lines on your diagram, m2 its insufficient because m could be a
it is doable. First draw lines bisecting negative or positive number. Together
each of the three angles in the triangle. however we know m is positive 2.64,
This forms six smaller 30:60:90 triangles. when n is 5.
The shortest side of each of those triangles
is equal to the radius of the circle. This 30. (B). If you sent up percents as simple
means that the radius on the circle is 3 , fractions they are easy to reduce and
.5 5 1 1
and the area of the circle is 3π . Plug all solve. = = = , from
of this informa tion into our initial 2.5 • 7 25 • 7 5 • 7 35
equation and we get the area of the here you can either ball park or divide.
portion described by the minor arc DE and
the line segments AD and AE is equal to 31. (B). In order to answer this question, we
need to know what proportion of the staff
9 3 − 3π
= 3 3 −π. belong to each division in order to know
3 how to weight the average. Statement 2
gives you that proportion, so its sufficient.
27. (E). The probability of getting an orange
and then a purple stone is 32. (D). Here, plugging in for n is key. If n =
 4  9  3 . 5, then you have the following expression:
   =
 16  15  20
r6 − r5 =
(− 1)5 = − 1 . This means that r
5
5 5
28. (E). This one is hard to sort out at first, is larger than r6 by 1/5. Eliminate answer
but if you plug in for a and b as specific
choices (A), (C), and (E). Now plug in n
values of x, the question begins to make
more sense. For example, if a = 2, b = 3, = 6 to see how r7 relates to everything.
and a + b = 5, then the answer choices When you do, you get
respond as follows:
r7 − r6 =
(− 1)6 = 1 . This means that r is
( ) (
(A) 5 2 + 1 ≠ 2 2 + 1 + 3 2 + 1) 6 6
7

(
(B) 2 5 ≠ 2 2 + 2 3 ) ( ) larger than r6 but less than r5 .
Etc.
33. (D). When you see the words "at least" in
a probability question, chances are it is
easier to find the probability of the
opposite happening and then subtract from
1. The opposite of at least one ball being
black is both balls being white. The
probability of drawing two white balls is
7 6 7
× = . Don't forget to subtract that
10 9 15
from 1!

12
The Princeton Review Management, LLC 2002
34. (D). This looks scary, but if you factor
out your common factors from each term,
it gets easier:
( )
2 74 − 2 70 = 2 70 2 4 − 1 = 2 70 (15 ) , so the
largest prime factor is 5.

35. (B). The total number of possible pin


numbers are 10 x 10 x 10 = 1000. If all
but 327 of them are in use, that means that
there are 673 club members.

36. (C). This one is all about exponents:


0.08 −5 0.04 4 (4 × 10 )
−2 4

= = =
0.04 −4 0.085 (8 × 10 −2 )5

(2 ) × 10
2 4 −8

= 2 8−15 × 10 −8− (− 10) =


(2 ) × 10
3 5 − 10

102 25
=
27 32

37. (A). If you square both sides in order to


rid yourself of the radical, you get x 2 =
6xy – 9y2 , you should notice this is a
quadratic equation. Reshuffle the equation
to standard form and factor and you get
(x – 3y)(x – 3y) = 0 and therefore
(x – 3y) = 0.

13
The Princeton Review Management, LLC 2002

Вам также может понравиться